0 Daumen
655 Aufrufe

Aufgabe:

Beweisen oder widerlegen Sie folgende Aussagen wie üblich mit einem genauen, kleinschrittigen Beweis.

\( (a) \) Die Folge \( \left(a_{n}\right)_{n \in \mathbb{N}}:=\left(2^{n}+2(-2)^{n-1}\right)_{n \in \mathbb{N}} \) ist beschränkt.

 könnte mir jemand diese Aufgabe bitte lösen. ich hab irgendwie kein Ansatzpunkt.

Avatar von

1 Antwort

0 Daumen
 
Beste Antwort

$$|2^n+2(-2)^{n-1}|=|2^n((-1)^{n+1}+1)|=\underbrace{2^n}_{>0}\underbrace{|((-1)^{n+1}+1)|}_{=0  \iff \,  n \text{ gerade}}\geq 0$$ Damit gilt genauer \(a_{2n}=0\) und \(a_{2n+1}=2^{n+1}\)

Hinweis:

Wie rumar richtig nachträgt, bedeutet das aber nicht, dass die Folge beschränkt ist. Denn nach oben hin, ist nicht beschränkt.

Avatar von 28 k

Somit ist die Folge also nicht beschränkt stimmst?

Doch, sie ist durch 0 nach unten beschränkt. Aus der Beschränktheit folgt aber keine Konvergenz, nur die Existenz einer konvergenten Teilfolge aus dem Satz von Bolzano-Weierstraß.

Dass der Betrag einer reellen Zahl nichtnegativ ist, dürfte wohl klar sein.

Eine Folge heißt erst dann beschränkt, wenn sie nach oben UND nach unten beschränkt ist !

Die hier angegebene Folge ist also nicht beschränkt (obwohl sie "nach unten beschränkt" ist).

Eine Folge heißt erst dann beschränkt, wenn sie nach oben UND nach unten beschränkt ist !

Da hast du recht!!!

Ein anderes Problem?

Stell deine Frage

Willkommen bei der Mathelounge! Stell deine Frage einfach und kostenlos

x
Made by a lovely community